Differential equations with eigenvalues.

Click For Summary
The discussion revolves around solving a differential equation using eigenvalues and eigenvectors. The eigenvalues obtained from the matrix are 3 and 4, leading to corresponding eigenvectors of [1, 1] and [2, 3]. The general solution is formed by combining these eigenvectors, resulting in x(t) = C1 * e^(3t) * [1, 1] + C2 * e^(4t) * [2, 3], where C1 and C2 are constants. Participants emphasize the importance of verifying that the solutions satisfy the original differential equation. The conversation also touches on the need for a calculator to check solutions for similar problems.
WendysRules
Messages
35
Reaction score
3

Homework Statement


Find all solutions of the given differential equations: ## \frac{dx}{dt} =
\begin{bmatrix}
6 & -3 \\
2 & 1
\end{bmatrix} x ##

Homework Equations

The Attempt at a Solution


So, we just take the determinate of A-I##\lambda## and set it equal to 0 to get the eigenvalues of 3, and 4. Then, we plug our eigenvalues one at a time into A-I##\lambda## and solve it to equal the 0 vector.

For 3, I get ##
\begin{bmatrix}
3 & -3 \\
2 & -2
\end{bmatrix} ## Which implies that ##x_1 = x_2## so I can just pick
##\begin{bmatrix}
1 \\
1
\end{bmatrix} ## to be my vector? This is the part I am confused about, I'm not sure that's what I actually do! Do I just say for ##\lambda## = 3, my solution is ## e^{3t} \begin{bmatrix}
1 \\
1
\end{bmatrix} ##

Similarly for ##\lambda = 4 ## i find my matrix to be
##
\begin{bmatrix}
2 & -3 \\
2 & -3
\end{bmatrix} ## which i see that ##2x_1=3x_2## this, i can just pick
##\begin{bmatrix}
2 \\
3
\end{bmatrix} ## Thus, my solution for ##\lambda = 4## is just ##e^{4t}
\begin{bmatrix}
2 \\
3
\end{bmatrix} ##

Thanks for all the help, I'm just unsure this is the method I should be going about to solve these types of problems! (Also, if anyone knows of an algebraic calculator in which I could check my solutions for these types of problems, I would also appreciate that)
 
Physics news on Phys.org
WendysRules said:

Homework Statement


Find all solutions of the given differential equations: ## \frac{dx}{dt} =
\begin{bmatrix}
6 & -3 \\
2 & 1
\end{bmatrix} x ##

Homework Equations

The Attempt at a Solution


So, we just take the determinate determinant of A-I##\lambda## and set it equal to 0 to get the eigenvalues of 3 and 4. Then, we plug our eigenvalues one at a time into A-I##\lambda## and solve it to equal the 0 vector.

For 3, I get ##
\begin{bmatrix}
3 & -3 \\
2 & -2
\end{bmatrix} ## Which implies that ##x_1 = x_2## so I can just pick
##\begin{bmatrix}
1 \\
1
\end{bmatrix} ## to be my vector? This is the part I'm confused about. I'm not sure that's what I actually do! Do I just say for ##\lambda## = 3, my solution is ## e^{3t} \begin{bmatrix}
1 \\
1
\end{bmatrix}##
Yup, that's right.

Similarly for ##\lambda = 4 ## i find my matrix to be
##
\begin{bmatrix}
2 & -3 \\
2 & -3
\end{bmatrix} ## which i see that ##2x_1=3x_2## this, i can just pick
##\begin{bmatrix}
2 \\
3
\end{bmatrix} ## Thus, my solution for ##\lambda = 4## is just ##e^{4t}
\begin{bmatrix}
2 \\
3
\end{bmatrix} ##
You got the 2 and 3 backwards.

Thanks for all the help, I'm just unsure this is the method I should be going about to solve these types of problems! (Also, if anyone knows of an algebraic calculator in which I could check my solutions for these types of problems, I would also appreciate that)
So now that you have to two solutions, what's the general solution?
 
WendysRules said:

Homework Statement


Find all solutions of the given differential equations: ## \frac{dx}{dt} =
\begin{bmatrix}
6 & -3 \\
2 & 1
\end{bmatrix} x ##

Homework Equations

The Attempt at a Solution


So, we just take the determinate of A-I##\lambda## and set it equal to 0 to get the eigenvalues of 3, and 4. Then, we plug our eigenvalues one at a time into A-I##\lambda## and solve it to equal the 0 vector.

For 3, I get ##
\begin{bmatrix}
3 & -3 \\
2 & -2
\end{bmatrix} ## Which implies that ##x_1 = x_2## so I can just pick
##\begin{bmatrix}
1 \\
1
\end{bmatrix} ## to be my vector? This is the part I am confused about, I'm not sure that's what I actually do! Do I just say for ##\lambda## = 3, my solution is ## e^{3t} \begin{bmatrix}
1 \\
1
\end{bmatrix} ##

Similarly for ##\lambda = 4 ## i find my matrix to be
##
\begin{bmatrix}
2 & -3 \\
2 & -3
\end{bmatrix} ## which i see that ##2x_1=3x_2## this, i can just pick
##\begin{bmatrix}
2 \\
3
\end{bmatrix} ## Thus, my solution for ##\lambda = 4## is just ##e^{4t}
\begin{bmatrix}
2 \\
3
\end{bmatrix} ##

Thanks for all the help, I'm just unsure this is the method I should be going about to solve these types of problems! (Also, if anyone knows of an algebraic calculator in which I could check my solutions for these types of problems, I would also appreciate that)

You are correct.

However, when in doubt, check to see if your alleged "solutions" actually satisfy the DE---a step you should always do, anyway, as part of good working practice.
 
What you want to do is merge those two eigenvectors into a matrix, and left-multiply it by the solution to the diagonal system of differential equations to get the general solution for your original matrix.
 
vela said:
So now that you have to two solutions, what's the general solution?

Should just be the two of the added together? So, ## x(t) = C_1 e^{3t} \begin{bmatrix}
1 \\
1
\end{bmatrix} + C_2 e^{4t}
\begin{bmatrix}
3 \\
2
\end{bmatrix} ## where C is a constant.

Ray Vickson said:
However, when in doubt, check to see if your alleged "solutions" actually satisfy the DE---a step you should always do, anyway, as part of good working practice.

Do I check them vs the original equations, or just the new "eigenedited" matrices? I.E for ## \lambda = 4## I'd use the matrix ##
\begin{bmatrix}
2 & -3 \\
2 & -3
\end{bmatrix} ## and multiple it by
##\begin{bmatrix}
3 \\
2
\end{bmatrix} ## to get
##\begin{bmatrix}
6+-6 \\
6+-6
\end{bmatrix} ## =
##\begin{bmatrix}
0 \\
0
\end{bmatrix} ## Thus it works.
 
WendysRules said:
Should just be the two of the added together? So, ## x(t) = C_1 e^{3t} \begin{bmatrix}
1 \\
1
\end{bmatrix} + C_2 e^{4t}
\begin{bmatrix}
3 \\
2
\end{bmatrix} ## where C is a constant.
Yes, that's correct.

Do I check them vs the original equations, or just the new "eigenedited" matrices?
You want to see if your solution ##x(t)## satisfies the differential equation, so plug it into the differential equation.
 
Question: A clock's minute hand has length 4 and its hour hand has length 3. What is the distance between the tips at the moment when it is increasing most rapidly?(Putnam Exam Question) Answer: Making assumption that both the hands moves at constant angular velocities, the answer is ## \sqrt{7} .## But don't you think this assumption is somewhat doubtful and wrong?

Similar threads

  • · Replies 1 ·
Replies
1
Views
2K
Replies
19
Views
1K
Replies
3
Views
2K
Replies
2
Views
1K
Replies
10
Views
2K
  • · Replies 10 ·
Replies
10
Views
2K
  • · Replies 2 ·
Replies
2
Views
1K
  • · Replies 2 ·
Replies
2
Views
2K
  • · Replies 4 ·
Replies
4
Views
3K
Replies
2
Views
2K